Find the area of the triangle with vertices A(-3,2), B(1,-2), and c(1,3)

Find The Area Of The Triangle With Vertices A(-3,2), B(1,-2), And C(1,3)

Answers

Answer 1

Answer:

[tex]A(-3,2)\: B(-1,-2)\: C(1,3)[/tex]

[tex]A=1/2[x_1(y_2-y_3)+x_2(y_3-y_1)+x_3(y_1-y_2)][/tex]

[tex]=1/2[-3(-2-3)(+1(3-2)+1(2-(-2))][/tex]

[tex]=1/2(15+1+4)[/tex]

[tex]=1/2(20)[/tex]

[tex]=10[/tex]

[tex]ANSWER: 10 ~units^{2}[/tex]

------------------------------

hope it helps...

have a great day!!

Answer 2

The area of triangle with vertices A(-3,2), B(1,-2), and c(1,3) is 12 units^2.

What is area of triangle?

The area of a triangle is defined as the total space occupied by the three sides of a triangle in a 2-dimensional plane. The basic formula for the area of a triangle is equal to half the product of its base and height, i.e., A = 1/2 × b × h.

The green line represents the height of the triangle = 4 units

The red line represents the base of the triangle = 6 units

Area of the triangle = 1/2 x 6 x 4 = 12 units^2

Learn more about area of triangle here

https://brainly.com/question/19305981

#SPJ2

Find The Area Of The Triangle With Vertices A(-3,2), B(1,-2), And C(1,3)

Related Questions

In In 5x + In In (x - 1) = 2

Answers

Answer:

exact form: x=-1/2

decimal form: x=-0.5

Someone please help me ASAP

Answers

Step-by-step explanation:

a vector multiplied by a scalar is equal to it's image. The expression above gives an equation and after solving, it gives you the image

plz help me with this math and also explain

Answers

Step-by-step explanation:

[1]

SI = $250Rate (R) = 12[tex] \sf \dfrac{1}{2}[/tex] %Time (t) = 4 years

[tex]\longrightarrow \tt { SI = \dfrac{PRT}{100} } \\ [/tex]

[tex]\longrightarrow \tt { 250 = \dfrac{P \times 12\cfrac{1}{2} \times 4}{100} } \\ [/tex]

[tex]\longrightarrow \tt { 250 = \dfrac{P \times \cfrac{25}{2} \times 4}{100} } \\ [/tex]

[tex]\longrightarrow \tt { 250 = \dfrac{P \times 25 \times 2}{100} } \\ [/tex]

[tex]\longrightarrow \tt { 250 = \dfrac{P \times 50}{100} } \\ [/tex]

[tex]\longrightarrow \tt { 250 \times 100 = P \times 50} \\ [/tex]

[tex]\longrightarrow \tt { 25000 = P \times 50} \\ [/tex]

[tex]\longrightarrow \tt { \dfrac{25000}{50} = P } \\ [/tex]

[tex]\longrightarrow \underline{\boxed{ \green{ \tt { \$ \; 500 = P }}}} \\ [/tex]

Therefore principal is $500.

__________________

[2]

2/7 of the balls are red.3/5 of the balls are blue.Rest are yellow.Number of yellow balls = 36

Let the total number of balls be x.

→ Red balls + Blue balls + Yellow balls = Total number of balls

[tex]\longrightarrow \tt{ \dfrac{2}{7}x + \dfrac{3}{5}x + 36 = x} \\ [/tex]

[tex]\longrightarrow \tt{ \dfrac{10x + 21x + 1260}{35} = x} \\ [/tex]

[tex]\longrightarrow \tt{ \dfrac{31x + 1260}{35} = x} \\ [/tex]

[tex]\longrightarrow \tt{ 31x + 1260= 35x} \\ [/tex]

[tex]\longrightarrow \tt{ 1260= 35x-31x} \\ [/tex]

[tex]\longrightarrow \tt{ 1260= 4x} \\ [/tex]

[tex]\longrightarrow \tt{ \dfrac{1260 }{4}= x} \\ [/tex]

[tex]\longrightarrow \underline{\boxed{ \tt { 315 = x }}} \\ [/tex]

Total number of balls is 315.

A/Q,

3/5 of the balls are blue.

[tex]\longrightarrow \tt{ Balls_{(Blue)} =\dfrac{3 }{5}x} \\ [/tex]

[tex]\longrightarrow \tt{ Balls_{(Blue)} =\dfrac{3 }{5}(315)} \\ [/tex]

[tex]\longrightarrow \tt{ Balls_{(Blue)} = 3(63)} \\ [/tex]

[tex]\longrightarrow \underline{\boxed{ \green {\tt { Balls_{(Blue)} = 189 }}}} \\ [/tex]

Which point is in the solution set of this system inequalities?

A. (0,0)

B. None of these

C. (5,1)

D. (3,7)

Answers

Answer:

B

Step-by-step explanation:

To find which ordered pairs are solutions to the inequalities we can simply plug in the x and y values of the ordered pairs into the inequalities and if the equation is true for both inequalities then the ordered pair is a solution to the inequalities.

For (0,0)

x = 0

y = 0

y > x + 5

Substitute 0 for y and x

0 > 0 + 5

Simplify right side

0 > 5

The inequality is not true as 5 is greater than 0, not less than. So immediately we can eliminate answer choice A.

For (5,1).

x = 5

y = 1

y > x + 5

Substitute 5 for x and 1 for y

1 > 5 + 5

Simplify right side

1 > 10

Again, the equation is not true as 1 is not greater than 10. This means that c cannot be the answer

For (3,7)

x = 3

y = 7

y > x + 5

Substitute 3 for x and y for 7

7 > 3 + 5

Simplify right side

7 > 8

7 is not greater than 8 meaning that (3,7) cannot be a solution to the inequalities

None of the ordered pairs created true equations hence the answer is B

find the volume of the rectangular prism. plz answer this lol

Answers

Answer:

.....how when the dimensions are not even clear lol

Answer:

48 cm³

Step-by-step explanation:

the volume of a rectangular prism= length × breadth × height

= 8× 3 × 2

= 48 cm³

if f(x) = 2x/7 +4, which of the following is the inverse of f(x)?

Answers

Answer:

The answer is C

Step-by-step explanation:

Switch x and y

[tex]x = \frac{2y}{7} + 4[/tex]

Solve for y

[tex]y = \frac{7(x - 4)}{2} [/tex]

t=29pi/6
1. find the reference number
2. find the point on the unit circle
3. 6 trig functiond

Answers

the correct answer is C....I hope

On a coordinate plane, a line goes through (negative 3, negative 4) and (3, 0).
What are the necessary criteria for a line to be perpendicular to the given line and have the same y-intercept?

The slope is Negative three-halves and contains the point (0, 2).
The slope is Negative two-thirds and contains the point (0, −2).
The slope is Three-halves and contains the point (0, 2).
The slope is Negative three-halves and contains the point (0, −2

Answers

Answer:

The slope is Negative three-halves and contains the point (0, 2).

Step-by-step explanation:

(-3,-4)(3,0)

M= -4/-6 = 2/3

⊥M = -3/2

0 = 2/3(3) + B

B=2

Drag the operator to the correct location on the image.
Which operation results in a binomial?

Answers

The correct answer is to drag The Plus sign (+)

What is an Operator?

This has to do with the use of symbols to denote mathematical equations such as addition, subtraction, etc.

Hence, we can see that the correct position to put the operator on the image to result in a binomial is to drag the plus sign (+) so that the equations can be solved,.

Read more about operators here:

https://brainly.com/question/25974538

#SPJ2

Answer:.

Step-by-step explanation:

3 questions only pls help gota finish tofday

Answers

5a) 3x3x13

b) 3x7x7

c)11x5x2x2

d)3x3x3x5x2

6a)2x2x2x131

7a)5x5x2x2

b) 3x2x17

c)5x2x2x2x2x2x2

d)5x5x5x3

e) 2x2x103

f)5x3x137

g) 2x2x2x2x2x2x2x2x2

h)5x5x7x7x3

One leg of a right triangle is 7 inches longer than the other leg, and the hypotenuse is 35 inches. Find the lengths of the legs of the triangle.

Answers

Answer: 21, 28

Step-by-step explanation:

Side #1 = xSide #2 = x + 7Hypotenuse = 35

Use the Pythagorean Theorem [tex]a^{2}+b^{2}=c^{2}[/tex]:

a = xb = x + 7c = 35

Substitute in the values & solve:

[tex]x^{2}+(x+7)^{2}=35^{2}\\x^{2}+x^{2}+14x+49=1225\\2x^{2}+14x+49-1225=0\\2x^{2}+14x-1176=0\\2(x^{2}+7x-588)=0\\2(x + 28)(x - 21)=0\\x_{1}=-28, x_{2}=21[/tex]

-28 is not a possible solution since you can't have negative inches...

a = x = 21b = x + 7 = 21 + 7 = 28c = 35

Twenty of the 50 digital video recorders (DVRS) in an inventory are known to be detective What is the probability that a randomly selected item is defective?​

Answers

Answer:

3/5

Step-by-step explanation:

Remember that odds for plus odds against must be 1. Therefore, 20 out of 50 of the DVRs are defective, so 30 out of 50 would work.

30/50

simplifies into 3/5 so that would be the probability of you picking out a DVR that works, and the probability of picking one that does not work would be the leftover 2/5 . Or, if you're answering with a percent, you would have a 60% chance of picking a working DVR.

I hope this helps, I know the explanation isn't really clear, but I can't really think of another way to explain it.

What is the slope of the line? What is the y-intercept of the line? y = -3x + 4

Answers

Answer:

slope= -3/1

y-intercept= 4

Answer:

m = -3

y intercept = 4

Step-by-step explanation:

The given equation of the line is ,

[tex]\implies y = -3x+4[/tex]

We know that the Standard equation of Slope Intercept Form of the line is,

[tex]\implies y = mx + c[/tex]

Where ,

m is slope c is y intercept

On comparing to the Standard form of the line we get ,

[tex]\implies Slope = -3 [/tex]

[tex]\implies y - intercept= 4 [/tex]

The sum of three consecutive odd numbers is 63. What are the numbers?​

Answers

19,21,23. Adding all of these consecutive odd numbers is equal to 63

Answer:

19, 21 and 23

Step-by-step explanation:

→ Make an algebraic expression for the 3 consecutive numbers

2x + 1, 2x + 3 and 2x + 5

→ Add the expressions together and make it equal to 63

2x + 1 + 2x + 3 + 2x + 5 = 63

→ Simplify

6x + 9 = 63

→ Minus 9 from both sides

6x = 54

→ Divide both sides by 6

x = 9

→ Resubstitute x = 9 into the 3 expressions

x = 9 into 2x + 1 is 19

x = 9 into 2x + 3 is 21

x = 9 into 2x + 5 is 23

log8-log4 ÷ log4-log2=





Answers

The answer is log(4)-1

What is 6.273 rounded to the nearest thousandths?

Answers

Answer:

6.270

Step-by-step explanation:

3 is below 5 so you just turn it into a 0. If it was 5 or above you would just add a number to the 7.

help help help pls :)

Answers

Answer:

[tex]opposite\approx 70.02[/tex]

Step-by-step explanation:

The triangle in the given problem is a right triangle, as the tower forms a right angle with the ground. This means that one can use the right angle trigonometric ratios to solve this problem. The right angle trigonometric ratios are as follows;

[tex]sin(\theta)=\frac{opposite}{hypotenuse}\\\\cos(\theta)=\frac{adjacent}{hypotenuse}\\\\tan(\theta)=\frac{opposite}{adjacent}[/tex]

Please note that the names ([tex]opposite[/tex]) and ([tex]adjacent[/tex]) are subjective and change depending on the angle one uses in the ratio. However the name ([tex]hypotenuse[/tex]) refers to the side opposite the right angle, and thus it doesn't change depending on the reference angle.

In this problem, one is given an angle with the measure of (35) degrees, and the length of the side adjacent to this angle. One is asked to find the length of the side opposite the (35) degree angle. To achieve this, one can use the tangent ([tex]tan[/tex]) ratio.

[tex]tan(\theta)=\frac{opposite}{adjacent}[/tex]

Substitute,

[tex]tan(35)=\frac{opposite}{100}[/tex]

Inverse operations,

[tex]tan(35)=\frac{opposite}{100}[/tex]

[tex]100(tan(35))=opposite[/tex]

Simplify,

[tex]100(tan(35))=opposite[/tex]

[tex]70.02\approx opposite[/tex]

Solve for x. Round to the nearest tenth, if necessary.

Answers

Answer:

11.3

Step-by-step explanation:

first we find angle F.

remember, all angles in a triangle always sum up to 180 degrees.

so,

F = 180 - 90 - 61 = 29 degrees

now we use the law of sines.

EF/sin(D) = ED/sin(F) = DF/sin(E)

DF = x

sin(E) = sin(90) = 1

5.5/sin(29) = x/1 = x

x = 11.3

sue has 18 pieces of candy
tony has 18 pieces of candy
sue then gives some to tony
sue then eats five of hers
tony eats half of his
write the expressions for the number of pieces candy sue and tony now have?

Answers

Answer:

Sue candy = 13 - x

Tony candy = 9 + 1/2x

Step-by-step explanation:

Sue candy = 18

Tony candy = 18

Let x = some candy gives to tony

Sue candy = 18 - x

Tony candy = 18 + x

sue then eats five of hers

Sue candy = 18 - x - 5

= 13 - x

tony eats half of his

Tony candy = 1/2(18 + x)

= 18/2 + x/2

= 9 + 1/2x

Expressions for the number of pieces candy sue and tony now have:

Sue candy = 13 - x

Tony candy = 9 + 1/2x

Find the measure of the missing angle using the exterior angle sum theorm.

Answers

Answer:

85°

Step-by-step explanation:

The exterior angle of a triangle is=sum of the opposite interior angles

So

? °=45°+40°

someone help me for this algebra task please

Answers

Answer:

200

Step-by-step explanation:

Substitute 15 for y

[tex] \frac{1}{5} x - \frac{2}{3} (15) = 30[/tex]

[tex] \frac{1}{5} x - 10 = 30[/tex]

[tex] \frac{1}{5} x = 40[/tex]

[tex]x = 200[/tex]

P(x) is a polynomial. here are a few values of p(x).
P(-5) = - 2
P(-3) = 6
P(3) = 7
P(5) = -1
What is the remainder when P(x) is divided by (x+5)?
What is the remainder when P(x) is divided by (x-3)?

Answers

Given:

Values of a polynomial P(x).

[tex]P(-5)=-2[/tex]

[tex]P(-3)=6[/tex]

[tex]P(3)=7[/tex]

[tex]P(5)=-1[/tex]

To find:

The remainder when P(x) is divided by (x+5).

The remainder when P(x) is divided by (x-3).

Solution:

If a polynomial P(x) is divided by (x-a), then the remainder is P(a).

If the polynomial P(x) is divided by (x+5), then the remainder is P(-5).

[tex]P(-5)=-2[/tex]

Therefore, the remainder is -2 when P(x) is divided by (x+5).

If the polynomial P(x) is divided by (x-3), then the remainder is P(3).

[tex]P(3)=7[/tex]

Therefore, the remainder is 7 when P(x) is divided by (x-3).

For the function G defined by G(x)=5x+3, find G(r+5)

Answers

Given function:

g(x) = 5x + 3

Find

g(r+5)

Substitute x with r = 5:

g(r + 5) = 5(r + 5) + 3 = 5r + 25 + 3 = 5r + 28

Answer:

G ( r + 5 ) = 5r + 28

Step-by-step explanation:

Given ;

G ( x ) = 5x + 3

To Find :-

G ( r + 5 )

Solution :-

plug r + 5 as x in the function.

G ( r + 5 ) = 5 ( r + 5 ) + 3

distribute 5

G ( r + 5 ) = 5r + 25 + 3

combine like terms

G ( r + 5 ) = 5r + 28

WILL GIVE BRAINLIEST AND 50 POINTS!
Determine the more basic function that has been shifted, reflected, stretched, or compressed.
m(x)= 2√x-5 -2

Answers

Answer: The function M if derived from the integral of the derivative of the function. This is valid by the fundamental theorem of calculus.

M(x)=43x32−7x+C

Step-by-step explanation: not 100 about this but it's what I got

In which section of the number line is 32−−√?

Answers

where's the number line?

maybe u can attach it at the comments:)

Answer:

Section B

Step-by-step explanation:

Identify a horizontal or vertical stretch or compression of the function by observing the equation of the function .

Answers

Bdbxbcjncnxndnsnns Identify a horizontal or vertical stretch or compression of the function by observing the equation of the function .hdbdbebqb

what is the length of segment DC? no links.

Answers

Answer:

33 units

Step-by-step explanation:

AB = BC

2x + 7 = 23

2x = 16

x = 8

DC = DA

DC = 4x + 1

DC = 4(8) + 1

DC = 33

one more question
(-8)+___=-17

Answers

Answer:

-9

Step-by-step explanation:

Answer:

-9

Step-by-step explanation:

-8 +___= - 17

___=-17 +8

___=-9

=-9

Please help. I don’t understand I rlly appreciate it if you help!

Answers

Hi there!

We know that the angles ∠B and ∠A are supplementary because they are both interior angles. Therefore:

180 = ∠A + ∠B

Rewrite:

180 = (6x - 48) + (4x + 38)

Combine like terms:

180 = 10x - 10

Solve for x:

190 = 10x

x = 19

Solve for ∠B by plugging in this value of x:

∠B = 4(19) + 38

∠B = 76 + 38

∠B = 114

A number ending in ___ is never a perfect square. ​

Answers

Answer:

2, 3, 7 or 8

Step-by-step explanation:

Other Questions
Help please? thank you Write a phrase in words to match each expression.5+3 n A box that is sliding across the floor experiences a net force of 10.0 N. If the box has a mass of 1.50 kg, what is the resulting acceleration of the box g A 3-year bond has an 8.0% coupon rate and a $1,000 face value. If the yield to maturity on the bond is 10%, calculate the price of the bond assuming that the bond makes semiannual coupon payments. Ling must spend no more than $40.00 on decorations for a party. She has spent $10.00 on streamers and wants to buy bags of balloons as well. Each bag of balloons costs $2.50. The inequality below represents x, the number of bags she can buy given the spending limit and how much she has already spent on streamers.10 + 2.5 x less-than-or-equal-to 40Which best describes the number of bags of balloons she can buy? Describe Effect of population growth on air pollution What is electrostatic attraction The passage suggests that in designing the experiment, Johnson relied on the fact that A) fungal hyphae warn beanstalks of danger if a nearby plant is attacked. B) wasps are harmful to the ongoing existence of broad bean plants. C) broad bean plants release noxious chemicals to ward off infestation. D) aphids are able to withstand the aggressive maneuvers used by wasps. A(-4,-2), B(-1,-1), C(-1,-4)what are the coordinate of triangle ABC when reflected over the line y=-x?A). A'(-2,4), B'(1,1), C(-4,1)B). A'(2,4), B'(1,1), C(4,1)C). A'(4,2), B'(1,2), C'(2,1)D). A'(2,4), B'(1,-1),C' (4,-1) Whats the equation of the line that passes through the point (-4,4) and has a slope of 3/4 Switched Ethernet, similar to shared Ethernet, must incorporate CSMA/CD to handle data collisions. True False Choose the best graph that represents the linear equation:y + 3 = 0Graph AOn a coordinate plane, a line goes through (0, 3) and (1, 3).Graph BOn a coordinate plane, a line goes through (negative 3, 0) and (negative 3, 1).Graph COn a coordinate plane, a line goes through (0, negative 3) and (1, negative 3).Graph DOn a coordinate plane, a line goes through (0, 0) and (1, negative 3).a.Graph Ac.Graph Cb.Graph Bd.Graph DPLEASE HELP!!! Please select the best answer from the choices providedABCD Amy has 2$, Jack has 3 times as much as Amy. Catherine has twice as much as Jack. How much does Catherine have? M H To determine the number of deer in a game preserve, a conservationist catches 412 deer, tags them and lets them loose. Later, 316 deer are caught, 158 of them are tagged. How many deer are in the preserve? Please answer fast!!!!!!The sum of the reciprocals of two consecutive even integers is 9/40. This can be represented by the equation shown.1/x+1/x+2=9/40Use the rational equation to determine the integers. Show all work. what is the example of comparision with as...as Describe fully the single transformation that maps a onto shape b Calculate the vapor pressure (in torr) at 298 K in a solution prepared by dissolving 46.8 g of the non-volatile non-electrolye glucose in 117 g of methanol. The vapor pressure of methanol at 298 K is 122.7 torr. Enter your answer to 2 decimal places. What is the percent yield of the reaction below if 84.0 grams of Al2O3(s) is recovered from a reaction whose theoretical yield of Al2O3(s) is 104 grams?4 Al(s) + 3 O2(g) 2 Al2O3(s) A quality control expert at Glotech computers wants to test their new monitors. The production manager claims they have a mean life of 82 months with a standard deviation of 7 months. If the claim is true, what is the probability that the mean monitor life would be greater than 83.8 months in a sample of 71 monitors